Sie sind auf Seite 1von 11

EXERCISE # 1

Q.1

Q.7

Which of the following is a vector ?


(A) Energy
(B) Power
(C) Force
(D) Mass

Q.8

Which of the following quantities is a scalar ?


(A) Intensity of gravitational field
(B) velocity
(C) charge on a proton
(D) Acceleration

Q.9

For the fig. (A) A + B = C


(B) B + C = A
(C) C + A = B
(D) A + B + C = 0

Q.10

Two forces of 4 dyne and 3 dyne act upon a


body. The resultant force on the body can
only be (A) more than 3 dynes
(B) more than 4 dynes
(C) between 3 and 4 dynes
(D) between 1 and 7 dynes(including 1 dyne
& 7 dyne)

Q.11

In the adjoining vector diagram,

d
(x4 2 sin x + 3 cos x)
dx
(A) 4x3 2 cos x + 3 sin x
(B) 3x2 + 2 cosx + 3 sin x
(C) 4x3 + 2 cosx 3 sin x
(D) 4x3 2 cos x 3 sin x

Q.2

d ( x 2 1)
dx x 1
x 2 2x 1
(A)
( x 1) 2

(C)

Q.3

x 2 2x 1
x 1

xy = c2, then
(A)

Q.4

x
y

(D)

(B)

y
x

(C)

x = at2 ; y = 2at, then

dy
dx

(A) t

(B)

x
y

(D)

1
t
(D) none

(1 x )
(A)

x 2 2x 1
( x 1) 2

dy
dx

(C) 1
Q.5

x 2 2x 1
(B)
( x 1) 2

y
x

what is the angle between A

x dx

and B ? (Given: C = B/2).

B
(A) 30
(B) 60

(C) 120
A
(D) 150

2 3/2 2 5/2
x +
x +C
3
5

2
2 5/2
(B) x3/2 +
x +C
3
5
(C)

2 3/2 2 5/2
x
x +C
3
5

(D) +

2 3/2 2 5/2
x
x +C
3
5

/ 2

Q.6

(sin x cos x )dx


0

(A) 2

(B) 1

(C) 3

(D) 4

CAREER POINT, CP Tower, Road No.1, IPIA, Kota (Raj.), Ph: 0744-3040000

ESSENTIAL MATHEMATICS & VECTOR

21

Q.12

The magnitude of the scalar product of these


two vectors is (A) 20
(B) 23
(C) 5 33
(D) 26

The magnitude of vectors A , B and C

are respectively 12, 5 and 13 units and

= C , then the angle

A B

Q.20

between A and B is Q.13

Q.14

Q.15

(A) 0
(B)
(C) /2 (D) /4
A child pulls a box with a force of 200N at an
angle of 30 above the horizontal. Then the
horizontal and vertical components of the
force are F
(A) 173 N, 100N
30
(B) 86.6N, 100N
(C) 100N, 86.6N
(D) 100N, 0N
A displacement vector, at an angle of 30 with
y-axis has an x-component of 10 units. Then
the magnitude of the vector is (A) 5.0
(B) 10
(C) 11.5
(D) 20

Q.16

Q.17

A B
sin

(C)

Q.21

A B
AB sin

(D)

A B
AB sin

If the displacement of a particle is given by

1 2
t + 4 t ) m then the velocity and
2
acceleration at t = 4s are v = 5 m/s and a = 7/8
m/s2.
S=(

Q.22

The maximum and minimum magnitudes of


the resultant of two given vectors are 17 and
7 respectively, If these two vectors are at right
angles to each other the magnitude of their
resultant is 13.

Q.23

(x

Q.24

Three forces start


acting simultaneously
on a particle moving

A vector perpendicular to (4
i 3 j ) is
(A) 4
(B) 7 k
i + 3 j
(D) 3
i 4 j

Angle between the vectors (


i + j ) and
(A) 90

Q.19

(B)

A.B

(
i j ) is -

Q.18

A B

(A)

(B) 4
(D) 10

(C) 6
i

unit vector perpendicular to both A and B


?

The projection of a vector r = 3


i + j + 2
on the x-y plane has magnitude k
(A) 3
(C) 14

Two vectors A and B are inclined to each


other at angle , which of the following is a

(B) 0

(C) 180 (D) 60

Angle that the vector A = 2


i + 3 j makes
with y-axis is (A) tan1 3/2

(B) tan1 2/3

(C) sin1 2/3

(D) cos1 3/2

Consider two vectors

and F = 3 j + 4 k
i +5k
F1 = 2
2

CAREER POINT, CP Tower, Road No.1, IPIA, Kota (Raj.), Ph: 0744-3040000

with

5) 3

dx is equal to.........................

velocity

v.

These forces are


represented in magnitude and direction by the
three sides of a triangle ABC as shown. Then
the velocity of particle will ..........
ESSENTIAL MATHEMATICS & VECTOR

22

Q.25

CAREER POINT, CP Tower, Road No.1, IPIA, Kota (Raj.), Ph: 0744-3040000

A truck travelling due north at 20 m/s turns


west and travels with same speed. Then the
change in velocity is ..........

ESSENTIAL MATHEMATICS & VECTOR

23

EXERCISE # 2
Q.7

The resultant of A and B makes an angle


with A and with B -

Q.1

Q.2

d
sin (log x)
dx
(A) cos (log x)

(B) log (cos x)

(C) x cos (log x)

(D)

Q.8

(B) < if A < B

(C) < if A > B

(D) < if A = B

2x 1

(2x2

(2x2

1)1/2

(B) 2x

1)1/2

(D)

(2x2

(2x2

(B) B may be a null vector

1)1/2

(C) A
may be to
B

+ 1)1/2

Q.4

If I =

(1 x ) 3
x

(D) A is || to B
dx ; then I =

(A) 2 x + 2x3/2

6 5/2 2 7/2
x x +C
5
7

(B) 2 x + 2x3/2 +

6 5/2 2 7/2
x + x +C
5
7

(C) 3

3/2
x + 3x

6 6/5 2 7/2
x x +C
5
7

(D) 3

6 6/5 2 7/2
3/2
x x +C
x 3x +
5
7

1 2 sin x
dx; then I is equal to
cos 2 x
(A) tan x + sec x + C (B) tan x sec x + C
(C) tan x 2 sec x + C (D) tan x + 2 sec x + C
I=

(A) A may be a null vector

(A) 2x

Q.3

If | A + B | = | A B |, which of the
following options is not true ?

d
dx
(C)

cos(log x )
x

(A) <

Q.9

Resultant of which of the following may be


equal to zero ?
(A) 10N; 10N; 30 N
(B) 10N; 20N; 30N; 40N
(C) 5N; 10N; 20N; 40N
(D) none of these

Q.10

A body is constraint to move only along


Y-direction under the action of a force
newton. If the body
F = 2
i + 15 j + 6 k
moves a distance of 10m then the work done
is (A) 150 J (B) 190 J (C) 163 J (D)185 J

Q.11

Let A = A cos
i + A sin j be any

Q.5

y=

vector. Another vector B which is normal to


1 sin 2 x

dx ; y is equal to

A is

(A) sin x cos x + C


(B) sin x + cos x + C
(C) 2 sin x cos x + C
(D) 2cos x sin x + C

Q.6

(A) B cos
i + B sin
(B) B sin
i + B cos
(C) B sin
i B cos

j
j
j

(D) A cos
i A sin j
Q.12

dx

(A) 1

(B) 0

(C)

(D) none of these

If A . B = | A B |, then angle

between vector A and B is (A) 0


(B) /4
(C) /2 (D)

Q.13

Which of the following is independent of


choice of coordinate system (A) P + Q + R
(B) (Px + Qx + Rx)
i
(C) (Px
i + Qy

Q.14

+ Rz) (D) all of these

3 and Q
The vectors P =
=
i a + j a + k

i a j 2 k

(A) 0

(B) 1

Q.20

(C) 2

(D) 3

(B)
(C)
(D)

having a magnitude equal to 3 is


(A) 3 k

Q.16

(B)

(
i + j )

) (D)
(
i +2k

(
i + j )

Q.21

direction is perpendicular to minimum force,


then the force are (C) 4 N & 12 N

(D) 2 N & 14 N

The two vectors A and B are drawn from

a common point and C = A B , then

angle between A and B is(A) 90 if C2 = A2 + B2


(B) greater than 90 if C2 < A2 + B2
(C) greater than 90 if C2 > A2 + B2
(D) less than 90 if C2 > A2 + B2

16N. If the resultant force is 8N and its

(B) 8 N & 8 N

| A B | may be smaller than | A B |

| A B | may be bigger than A + B

| A B | may be smaller than A B

| A B | may be smaller than both A &

The sum of two forces acting at a point is

(A) 6 N & 10 N

Choose the correct statement(A)

vector C perpendicular to both A and B and

B are non zero non-collinear vector)


(A) A
(B) B
(C)
AB

(D) A B

If A =
i + j and B =
i j , then a

(C)

A B is perpendicular to (where A and

are perpendicular to each

other. The positive value of a is -

Q.15

Q. 19

Q.22

Let C A B .

(A) | C | is always greater than | A |


Q.17

) newton acts on
A force of (10
i 3 j + 6 k
a body of mass 100 g and displaces it from (6
i +

(C) C is always equal to A + B


(D) C is never equal to A + B

metre. The work done is


(B) 121 J

(C) 361 J (D) 1000 J


Q.23

Q.18

If vector P , Q & R have magnitude 5,

12 and 13 units and P + Q = R

|C |<|B|

) meter to (10
)
5 j 3 k
i 2 j + 7 k
(A) 21 J

(B) It is possible to have | C | < | A | and

angle between Q and R is (A) cos1

5
12

(B) cos1

5
13

(C) cos1

12
13

(D) cos1

2
13

the

The x-component of the resultant of several


vectors (A) is equal to the sum of the x-components
of the vectors
(B) may be smaller than the sum of the
magnitudes of the vectors
(C) may be greater than the sum of the
magnitudes of the vectors
(D) may be equal to the sum of the
magnitudes of the vectors

Q.24

Three coplanar forces F1 , F2 and F3 are


acting simultaneously on a particle. Column I
contains different operation between forces
and Column II contains their magnitude.
Match them.
Column-I

Column-II

(A) F1 F2 F3

(B) F1 F2 F3
(C) F1 F2 F3

(D) | F3 .( F1 F2 ) |
Q.25

If

A 2i 3j k

(P) 0
(Q) 20
(R) 20 2
(S) 200
and

then Column I

(A) | A B |
(B) | A B |

(C) A . B

(D) | A B |

B i 2j 2k

Column II
(P) 11
(Q) 6
(R) 35
(S)

90

EXERCISE # 3
Q.1

Eliminate from the following equations :

Q.10

x-coordinate.
Find the slope of the tangent to curve
x3 + 3xy + y3 = 1 at (1, 1).

Q.11

Integrate w.r.t. to 'x'

(i) x = tan y = sec


(ii) x = a cos , y = b sin

Q.2

Q.3

Prove the following trignometrical identities

cos
cos
+
= 2 sec
1 sin
1 sin

(a)

(b)

Prove :

(c) sin x cos x

(d)

tan A + cot A =

Q.4

sec 2 A cos ec 2 A

Q.12

Differentiate w.r.t. 'x'


(a)

(c) x3 +

1
+8
x3

Q.6

Q.7

Q.8

(c)

Q.13

Q.14

A particle moves along the curve ; 6y = x3 + 2.


Find the points on the curve at which the
y-coordinate is changing 8 times as fast as the

GMm
dx
x2

/ 2

(b)

cos x dx

/ 2

1 cos x dx

The changes in a function y and the


independent variable x are related as

(a) Find the area bounded by the curve y = e x,


the x-axis and y-axis.
(b) Find the area enclosed by the curve y =
sin x and the x-axis between x = 0 and
x = .

Q.15

Suppose a is a vector of magnitude 4.5 unit


due north. What is the vector (a) 3a
(b) 4a

Q.16

Refer to figure. Find


(a) the magnitude
(b) x and y
components and
(c) the angle with
the x-axis of the

circuit is given by i = i0et/RC where i0 ; R and

Q. 9

x a2

dy
= x2. Find y as a function of x.
dx

The electric current in a charging R C


C are constant parameters of the circuit and t
is time. Find the rate of change of current at
(a) t = 0
(b) t = RC
(c) t = 10 RC

x
2

Find two numbers whose sum is 24 and


whose product is as large as possible.

A balloon, which always remains spherical,


has a variable radius. Find the rate at which
its volume is increasing w.r.t its radius when
the radius is 7cm.

(ax b) 2

/ 2

(d) sin x3

Amongst all pairs of positive numbers with


product 256. Find those whose sum is the
least.

(ax b) 2

(e) 4 x 3 5
Q.5

(a)

Evaluate the following integral :


R

(b)

resultant of OA,
BC and DE.
Q.17

Let A and B be the two vectors of magnitude


10 unit each. If they are inclined to the x-axis
at angles 30 and 60 respectively, find the
resultant.

Q.18

A vector A makes an angle of 20 and B


makes an angle of 110 with x-axis. The
magnitude of these vectors are 3m and 4m
respectively. Find the resultant.

Q.19

A vector has component along the x-axis


equal to 25 unit and along the y-axis equal to
60 unit. Find the magnitude and direction of
the vector.

Q.20

A spy report about a suspected car read as


follows. The car moved 2.00km east, made a
perpendicular left turn, ran for 500m, made a
perpendicular right turn, ran for 4.00 km and
stopped. Find the displacement of the car.

Q.21

Add vectors A, B and C each having


magnitude of 100unit and inclined to the xaxis at angles 45, 135 and 315 respectively.

Q.22

A carrom board (4ft 4ft square) has the


queen, at the centre. The queen hit by the
striker moves to the front edge, rebounds and
goes into the hole behind the striking line.
Find the magnitude of displacement of the
queen.
(a) from the centre to the front edge
(b) from the front edge to the hole
(c) from the centre to the hole

the components of the displacement


vector.
Q.24

The magnitudes of vectors OA, OB and OC


in figure are equal.
Find the direction of
OA + OB OC.

Q.25

The sum of three


vectors shown in
figure is zero. Find
the magnitude of
the vectors OB
and

Q.26

A mosquito net over a 7ft x 4ft bed is 3ft


high. The net has a hole at one corner of the
bed through which a mosquito enters the net
it files and sits at the diagonally opposite
upper corner of the net.
(a) Find the magnitude of the displacement of
the mosquito.
(b) Taking the hole as the origin, the length of
the bed as the x-axis, its width as the yaxis and vertically up as the z-axis, write

The resultant
of vectors OA
and OB is
perpendicular
to OA . Find
the angle
AOB.

Q.27

Two vectors have magnitudes 2m and 3m.


The angle between them is 60. Find (a) the
scalar product of the two vectors, (b) the
magnitude of their vector product.

Q.28

and b = 3
(a) Let a = 2
i + 3 j + 4 k
i +4
j

. Find the angle between them.


+5k

and B = 4
(b) If A = 2
i + 3 j + 4 k
i +3
j

Q.23

OC

, find A B.
+2k

Q.29

On an open ground, a motorist follows a truck


that runs to his left by an angle 60 after every
500 m. Starting from a given turn specify the
displacement of the motorist at the third, sixth
and eighth turn. Compare the magnitude of
the displacement with the total path length
covered by the motorist in each case.

Q.30

and B =
If A = 2
i + 2 j + k
i + 6 j + 2
, then find
k

(i) the projection of A along B


(ii) the projection of B along A
Q.31

Three forces F1 , F2 and F3 are such that

F1 = F2 + F3 and their magnitudes are 5,


F1 and F3 .

Passage (Q.33 to Q.35)


An object is rotating on a horizontal circular
track with constant speed u0 in clock-wise
direction. Initially object was moving in east
direction.

If the position vector of a particle is given by

r = (4 cos 2t) i + (4 sin 2t) j + 6t k m.

Q. 33

4 and 3 respectively. Find the angle between

Q.32

Calculate its acceleration at t =

.
4
Q. 34

Q. 35

Change in velocity as object rotates by an


angle 90 is(A) zero
(B) 2u0
(C) 2 u0
(D) 3 u0
Direction of change in velocity in above
question is(A) south-east
(B) south-west
(C) south
(D) west
Change in velocity as objects rotates by an
angle 180 is(A) zero
(B) 2 u0 towards west
(C) 2 u0 towards west
(D) 2 u0 towards south

ANSWER KEY
EXERCISE # 1

21.

True

22.

True

23.

24.

remain unchanged

25.

20 2 south-west

I=

x7
+ 5x5 + 25x3 + 125x + C
7

EXERCISE # 2
PART-A

PART-B

PART-C
24. A P ; B R ; C Q ; D S

25. A S ; B P ; C Q ; D R

EXERCISE # 3
PART-A
x

1. (i) y2 x2 = 1 (ii)

1 3/2
x
2
5. 12, 12
4. (a)

7.

=1

(b) 2a (ax + b)3

196 cm2

9. ( 4, 31/3), (4, 11)


11. (a)

(c) 3x2 3x4

(d) cosx3. 3x2

(e) 6x2 (4x3 5)1/2

6. (16, 16)
8. (a)

i0
i0
i0
(b)
(c) 10
RC
eRC
e RC

10. 1

5 6/5
(ax b) 1
x + C (b)
+C
6
a

(c)

cos 2 x
1
+ C(d)
log (x2 + a2) + C
4
2

12. (a)

GMm
R

(b) 2

(c) 2

13. y =

x3
+C
3

14. (a) 1 (b) 2


15. (A) 27/2 unit north (B) 18 unit south
16. (a) 1.6 m

(c) tan1(1.32)

(b) 0.98 m and 1.3 m respectively

17. 20 cos 15

18. 5, tan = 4/3

20. 6.02, tan1 1/12

21. 100 unit at an angle of 45 to x-axis

22. (A)

2
4
10 ft (B)
10 ft (C)
3
3
1 3

24. tan =

27. (a) 3 (b) 3 3

2 1

19. 65, tan = 60/25

ft.

23. (a)

25. | OB | = 5, | OC | = 5 2
28. (a) cos1

38
1450

74

ft (b) 7ft; 4 ft, 3 ft

26. = cos1(2/3)

(b) 6
i + 12

6 k

29. (i) 1000 m, 1500 m; (ii) 0, 3000 m; (iii) 866 m, 4000 m


30. (i)

16
41

(ii)

16
3

31. = cos1(3/5)

32. 16 j m/s2

PART-B
33. (C)

34. (B)

35. (C)

Das könnte Ihnen auch gefallen